Query failed: connection to 172.31.3.4:9312 failed (errno=111, msg=Connection refused). PT1.S3.Q7 - game ducks at a western lake - 7Sage Forum

PT1.S3.Q7 - game ducks at a western lake

AmbitiousAmbitious Free Trial Member
edited December 2015 in Logical Reasoning 7 karma
Hello,

I don't understand why A is the correct answer. I chose C. Could someone please explain why the right answer is right and the wrong answers are wrong? Thank you

Comments

  • HibiscusHibiscus Free Trial Member
    82 karma
    "The greater the disparity in overall sex ratios, the greater the percentage of older male ducks in the population."
    The western lake contains 55% males, 45% females
    The eastern lake contains 65% males, 35% females

    A states that the percentage of adult males in the western lake is lower than the eastern lake, which is correct because the disparity in overall sex ratios is greater in the eastern lake.

    B is not the answer because the passage doesn't say anything about the percentage of young male ducks.

    C states the total number of male ducks is higher in eastern lake than the western lake, which is not necessarily true. There could be 100 ducks in the eastern lake and 500 ducks in the western lake, which means the eastern lake has 65 male ducks, and the western lake has 275 male ducks. A higher percentage does not equate to a higher number when you don't know the sample size.

    D discusses something that isn't even in the passage at all.

    E states that adult female ducks outnumber young female ducks, but we do not have information about this. Passage states that there are only slightly more males than females in the young population, and number of males in adult population greatly exceeds females. We need to know the proportions or size of the adult population compared to the young population in order to answer this question.
  • Q.E.DQ.E.D Alum Member
    edited December 2015 556 karma
    Either I'm a fool or they messed up this question.

    I posted a counter-example to it recently, but I think the moderators took it down because I linked a spreadsheet in my Google Drive.

    West
    Male: 38 Adult (38%), 17 Young, 55 Total
    Female: 29 Adult, 16 Young, 45 Total

    East
    Male: 34 Adult (34%), 31 Young, 65 Total
    Female: 5 Adult, 30 Young, 35 Total

    This example abides by the following rules laid out in the stimulus:
    - Western M:F is 55% and eastern M:F is 65%
    - Only slightly more males than females among young ducks
    - "Appreciably" more males than females among adults

    Yet it's clear that the eastern lake need not have more adult males, proportionally, than the western lake. Given a large enough young duck pop. at the eastern lake, afforded by a massive M:F disparity in the adults, you can get a 65/35 ratio without exceeding the western lake's % adult male pop.

    What did I miss?

  • runiggyrunruniggyrun Alum Inactive Sage Inactive ⭐
    2481 karma
    QED, I think your hypothetical for Eastern lake doesn't quite comply with the statement that "there are appreciably more males among adults than among the young". They tell us that not only do the adult males greatly exceed the adult females, but they also appreciably exceed the juvenile males. How much is "appreciably"? Apparently enough for the researchers to conclude that the difference in adult male percentages is sufficient to account for the difference between total male percentages (for your East example, 39old/26young males and 10old/25young females would do it). On a side note, these scenarios with a larger difference between the adult and juvenile males also look a bit more realistic in terms of adult females/total babies (10 female ducks might have had 50 total offspring, but it's a bit of a stretch that 5 female ducks had 61 offspring). Not that the LSAT cares, of course, but you can see that it doesn't take much of a stretch to reach numbers that make the scientists conclusion entirely plausible.
    Because this is not a flaw question, we can take everything in the passage, including the scientists inference as true, and without having to bother with any of the "raw" data, we can apply their conclusion to the answers and come up with A as correct.
  • Q.E.DQ.E.D Alum Member
    edited December 2015 556 karma
    Hey @runiggyrun, thanks for the response. I agree with you about the correct answer choice. It's clear they want us to choose that. I'm just saying that neither the answer nor the inference in the stim follows from what they give. And we can easily shift the numbers to accommodate your reading without making the inference right.
Sign In or Register to comment.